LSAT and Law School Admissions Forum

Get expert LSAT preparation and law school admissions advice from PowerScore Test Preparation.

 Administrator
PowerScore Staff
  • PowerScore Staff
  • Posts: 8917
  • Joined: Feb 02, 2011
|
#23127
Complete Question Explanation

Method of Reasoning. The correct answer choice is (B)

Due to the relatively complex structure of the curator's argument, it is important to understand it fully before proceeding to answer the question:
  • Premise (1): ..... Reliable tests show that the red pigment was applied after the painting had been completed

    Premise (2): ..... The red paint was not mixed in Veronese's workshop

    Sub. Conclusion: ..... An artist other than Veronese tampered with his painting after its completion

    Main Conclusion: ..... The decision to restore the cloak of the central figure in Veronese's painting from its present red to the green found underneath is fully justified
As is often the case with Method of Reasoning/Argument Part questions, it is crucial to distinguish between the "subsidiary" conclusion and the "main" conclusion of the argument, particularly since the test makers will often try to trick you by prefacing the subsidiary conclusion with a typical conclusion indicator ("hence") while providing no conclusion indicators for the main conclusion. This tactic underscores the importance of understanding the function of each statement.

Compare the two contender conclusions and check to see which one answers the question "why?" — whichever one does, this will be the subsidiary conclusion of the argument. In this instance, the decision to restore the cloak is explained by the observation that someone must have tampered with the painting. Therefore, tampering with the painting is not the main conclusion of the argument, even though two separate premises are used in its support. By answering the question "why is the decision to restore the color of the cloak justified?", the statement that someone else must have tampered with Veronese's painting provides a premise for the main conclusion. Because this claim is both supported by the initial two premises and in turn supports another statement, it is a subsidiary conclusion of the argument. Answer choice (B) is therefore correct.

Answer choice (A) As discussed above, the main conclusion is the first sentence of the stimulus and not the last one, despite the misleading conclusion indicator ("hence") provided here. This answer choice is incorrect.

Answer choice (B): This the correct answer choice. See discussion above.

Answer choice (C) This statement does not clarify a key term. This answer choice is incorrect.

Answer choice (D) There is no general position in the curator's argument that needs defending. This answer choice is incorrect.

Answer choice (E) While the main point is in the first sentence, the last one does not reiterate it. Rather, it provides a rationale for it, by explaining why the decision to restore the color of the cloak is justified. This answer choice is incorrect.
 yrivers
  • Posts: 68
  • Joined: Mar 15, 2017
|
#35747
The PowerScore textbook from class says that C is the correct answer; the explanation above says it's B. Can you please clarify which one is correct? If C is correct, could you please explain why A, B, and E are incorrect?

Thank you!
User avatar
 Jonathan Evans
PowerScore Staff
  • PowerScore Staff
  • Posts: 726
  • Joined: Jun 09, 2016
|
#35815
Hi, YRivers,

The correct answer is definitely B. My edition of the PowerScore text (published 2016) indicated that B is the correct response on page 7-130. Is there something different in your edition? Please let us know if there is a misprint! Thank you!
 rockchalkLSAT
  • Posts: 1
  • Joined: Aug 22, 2017
|
#38671
Jonathan Evans wrote:Hi, YRivers,

The correct answer is definitely B. My edition of the PowerScore text (published 2016) indicated that B is the correct response on page 7-130. Is there something different in your edition? Please let us know if there is a misprint! Thank you!

I have the 2017 version of the LRQTT book and my answer key also indicates that C is the correct answer. The questions reads:

Curator: The decision to restore the cloak of the central figure in Veronese's painting from its present red to the green found underneath is fully justified. Reliable x-ray and chemical tests show that the red pigment was applied after the painting had been completed, and that the red paint was not mixed in Veronese's workshop. Hence it appears likely that an artist other than Veronese tampered with Veronese's painting after its completion.
Art Critic: Cut in a copy of Veronese's painting made shortly after Veronese died, the cloak is red. It is highly unlikely that's copyist would have made so major a change so soon after Veronese's death.

The art critic's response to the curator would provide the strongest support for which one of the following conclusions?

A: The copy of Veronese's painting that was made soon after the painter's death is indistinguishable from the original.
B: No painting should be restored before the painting is rested with technologically sophisticated equipment
C: The proposed restoration will fail to restore Veronese's painting to the appearance it had at the end of the artist's lifetime
D The value of an artist's work is not necessarily compromised when that work is tampered with by later artists
E: Veronese di not originally intend the central figure's cloak to be green

Can you please clarify what the correct answer is? Thanks!
 Francis O'Rourke
PowerScore Staff
  • PowerScore Staff
  • Posts: 471
  • Joined: Mar 10, 2017
|
#38720
Hi Rockchalk,

It looks like you are on the wrong forum post. This is where you want to be: lsat/viewtopic.php?t=9261

This question was # 9 on Section 4 of the October 1996 LSAT. Both question #9 and question #10 referred to the same stimulus, so they are easy to mix up.

Question #9 was a Method of Reasoning, Argument Part Question. It asked for the role played by the assertion that a later artist tampered with the painting. The correct answer was choice (B)

Question #10 was a Must be True Question. It asked for a conclusion that the art critic's argument could support. The correct answer to this was choice (C).
 jessamynlockard
  • Posts: 42
  • Joined: Jan 15, 2018
|
#44840
With an answer choice like A, should we almost always interpret "main point" to be synonymous with the conclusion?
 Malila Robinson
PowerScore Staff
  • PowerScore Staff
  • Posts: 296
  • Joined: Feb 01, 2018
|
#44914
Hi Jessamyn,
It may be better to think of the main point and the main conclusion as being synonymous with each other. There may be multiple conclusions, and a sub-conclusion may not fully match with the main point.
Hope that helps!
-Malila
 ronibass
  • Posts: 15
  • Joined: Jun 18, 2019
|
#66544
I had a quick question. When the question stem addresses one person's argument and they are the first speaker, in the case the curator, is it even necessary to read the statement by the second speaker, the art critic? Is this a good strategy or no?
 James Finch
PowerScore Staff
  • PowerScore Staff
  • Posts: 943
  • Joined: Sep 06, 2017
|
#66770
Hi Roni,

This is no longer an issue with newer tests, as the two question stimulus was phased out back in the early 2000s. That said, you should always read the entire stimulus before moving on to the question stems, including in cases where multiple questions are attached.

Get the most out of your LSAT Prep Plus subscription.

Analyze and track your performance with our Testing and Analytics Package.